Ellus


These prisms are similar. Find the surface


area of the larger prism in decimal form.


5 m


7 m


Surface Area


90 m2


Surface Area = [? ] m2


please helppp. :)

Answers

Answer 1

If two prisms are similar, their corresponding dimensions are proportional.

Let's assume that the ratio of the corresponding lengths of the smaller prism to the larger prism is k:1, where k is a constant.

Then the ratio of the corresponding surface areas of the smaller prism to the larger prism is [tex](k^2):1[/tex], because the surface area of a prism is proportional to the square of its length.

In this problem, the surface area of the smaller prism is not given.

However, we can find the ratio of the corresponding lengths of the smaller prism to the larger prism using the fact that they are similar.

The height of the smaller prism can be found as follows:

[tex]7/5 = h/L[/tex]

where h is the height of the smaller prism and L is the length of the larger prism.

Solving for h, we get:

[tex]h = (7/5)L[/tex]

The ratio of the corresponding lengths of the smaller prism to the larger prism is 7:5.

The ratio of the surface areas of the smaller prism to the larger prism is:

[tex](7/5)^2 : 1 = 49/25 : 1[/tex]

We know that the surface area of the larger prism is [tex]90 m^2.[/tex]

Let's denote the surface area of the smaller prism by A. Then we can set up an equation:

(49/25)A = 90

Solving for A, we get:

A = (25/49) * 90 = 45/7 ≈ 6.4

The surface area of the smaller prism is approximately [tex]6.4 m^2.[/tex]

(Note: The units of the surface area are not provided for the smaller prism, so I assumed the same units as the larger prism.

To know more about area refer here

https://brainly.com/question/14759304#

#SPJ11


Related Questions

QUESTION IN PHOTO I MARK BRAINLIEST

Answers

The value of measure of arc QI is,

⇒ m QI = 94°

We have to given that;

⇒ m YS = 180°

⇒ m ∠QBI = 137°

Hence, We can formulate;

⇒ m ∠QBI = 1/2 (m YS + m QI)

⇒ 137 = 1/2 (180 + m QI)

⇒ 274 = 180 + m QI

⇒ m QI = 274 - 180

⇒ m QI = 94°

Thus, The value of measure of arc QI is,

⇒ m QI = 94°

Learn more about the angle visit:;

https://brainly.com/question/25716982

#SPJ1

A pet boarder keeps a dog-to-cat ratio of 5:2. If the boarder has room for 98 animals then how many of them can be dogs

Answers

The pet boarder can accommodate 70 dogs.

To determine the number of dogs

Let's calculate how many dogs and cats the pet boarding facility can hold using the ratio of dogs to cats, which is 5:2.

First, we can figure out how many parts there are in the ratio: 5 + 2 = 7.

This indicates that there are 7 equal components in the ratio, 5 of which are dogs and 2 of which are cats.

We need to multiply the result by the number of dog components (5) after dividing the total number of parts (7) into the 98 available locations to get the number of dogs:

Number of dogs = (5 / 7) * 98

Number of dogs = 70

Therefore, the pet boarder can accommodate 70 dogs.

Learn more about ratio here : brainly.com/question/26460724

#SPJ1

Ao



Del



5. An archway has vertical sides 10 feet high. The top of an archway can



be modeled by the quadratic function f(x) = -0. 5x2 + 10 where x is the



horizontal distance, in feet, along the archway. How far apart are the



walls of the archway? Round your answer to the nearest tenth of a foot.



Original content Copyright © by Houghton Mifflin Harcourt. Additions and changes to the original content are the responsibility of the instructor.



293

Answers

The walls of the archway are approximately 8.9 apart.

Find out the distance between the walls of the archway?

To find the distance between the walls of the archway, we need to find the horizontal distance where the function f(x) intersects the x-axis. This is because the archway's walls are vertical, and their distance apart is the same as the horizontal distance between the points where the archway meets them.

To find the x-intercepts of the function f(x) = -0.5x^2 + 10, we need to set f(x) = 0 and solve for x:

0 = -0.5x^2 + 10

0.5x^2 = 10

x^2 = 20

x = ±√20

Since the archway is a physical object, we can discard the negative value for x, which means the archway meets the walls at x = √20 feet.

To find the distance between the walls of the archway, we can double this value:

2√20 ≈ 8.94

Then it's concluded that the walls of the archway are approximately 8.9 feet apart.

Learn more about Function

brainly.com/question/5975436

#SPJ11

Suppose a homing pigeon is released on an island at point C, which is 9 mi directly out in the water from a point B on shore, Point B is 20 mi downshore from the pigeon's home loft at point A. Assume that a pigeon flying over water uses energy at a rate 1.29 times the rate over land. Toward what points downshore from A should the pigeon fly in order to minimize the total energy required to get to the home loft at A? Point S ismiles away from point A. (Type an integer or decimal rounded to three decimal places as needed.)

Answers

The pigeon should fly directly from point C to point B, then fly along the shoreline to a point 10.387 miles away from point A (rounded to three decimal places). This can be found using the principle of minimizing the total distance traveled, taking into account the different energy rates over water and land.

To minimize the total energy required for the homing pigeon to get to its home loft at Point A, we need to find the optimal point downshore, Point S, to fly to. Using the given information, we can set up a function for the total energy.

Let x be the distance from Point A to Point S. Then, the pigeon will fly x miles over land and the remaining distance, 20-x miles, downshore from Point B to Point S. The distance from Point C to Point S can be found using the Pythagorean theorem:

CS = sqrt((20-x)^2 + 9^2)

Since the pigeon uses energy at a rate 1.29 times over water compared to land, we can write the total energy function as:

E(x) = x + 1.29 * CS

Now we need to minimize this function. To do so, we can take the derivative of E(x) with respect to x and set it equal to zero:

dE(x)/dx = 0

By solving this equation for x, we will find the optimal distance downshore from Point A to Point S. Once you have the value of x, you can say that Point S is x miles away from Point A (rounded to three decimal places, as needed).

Visit here to learn more about Pythagorean theorem brainly.com/question/14930619

#SPJ11

The cost to produce x kilograms of whatchamacallits is given by the function C(x) = 50x + 1000 where Cix) is in hundreds of dollars. The revenue for the sale of x whatchamacallits is given by R(x) = 450x where R(x) is in hundreds of dollars. How many kilograms should be produced and sold to realize a maximum profit? What is that maximum profit?

Answers

The maximum profit will be realized by producing and selling 2.5 kilograms of whatchamacallits.

The maximum profit, in hundreds of dollars, will be $500.

To find the maximum profit, we need to first calculate the profit function P(x), which is the difference between the revenue and the cost functions:

P(x) = R(x) - C(x)
P(x) = 450x - (50x + 1000)
P(x) = 400x - 1000

To find the amount of kilograms that should be produced and sold to realize a maximum profit, we need to find the value of x that maximizes the profit function.

We can do this by taking the derivative of the profit function and setting it equal to zero:

P'(x) = 400
400 = 0

Since the derivative is a constant value, there is no critical point or inflection point. Therefore, the profit function is increasing at a constant rate, and the maximum profit will be achieved at the highest possible value of x.

To find that value, we can set the profit function equal to zero and solve for x:

P(x) = 400x - 1000 = 0
400x = 1000
x = 2.5

Therefore, the maximum profit will be realized by producing and selling 2.5 kilograms of whatchamacallits.

To find the maximum profit itself, we can substitute this value of x into the profit function:

P(2.5) = 400(2.5) - 1000 = 500

So the maximum profit, in hundreds of dollars, will be $500.

To learn more about profit function, refer below:

https://brainly.com/question/16866047

#SPJ11

Mara putting together pieces of string for an art project. She has a piece of string that is 30 inches, a piece that is 22 inches, and a piece that is 20 inches. Once she puts together the pieces, what will be the total length in feet?



Step 1 - What will be the total length of the string in inches?





Step 2 - How many feet is this equal to? (Inches -> Feet)





2. A water jug holds 300 ounces of water. The football team has 2 water jugs. How many cups of water will both water jugs hold altogether?



Step 1 - How many ounces do both water jugs hold?




Step 2 - How many cups is this equal to? (Ounces -> Cups



Please help me if you help me and explain all the answer I will give you brainiest!!!

Answers

The total length of the string in feet is 6 feet, and the combined capacity of both water jugs in cups is 75 cups.

What is the total length of the string, and how many cups of water can the two water jugs hold altogether?

Step 1: To find the total length of the string in inches, Mara needs to add the lengths of the three pieces of string:

30 inches + 22 inches + 20 inches = 72 inches

So the total length of the string in inches is 72 inches.

Step 2: To convert inches to feet, we need to divide the number of inches by 12 (since there are 12 inches in a foot):

72 inches ÷ 12 = 6 feet

Therefore, once Mara puts together the three pieces of string, the total length will be 6 feet.

Step 1: To find out how many ounces of water both water jugs hold altogether, we need to add the capacity of the two jugs:

300 ounces + 300 ounces = 600 ounces

So both water jugs together can hold 600 ounces of water.

Step 2: To convert ounces to cups, we need to divide the number of ounces by 8 (since there are 8 ounces in a cup):

600 ounces ÷ 8 = 75 cups

Therefore, both water jugs together can hold 75 cups of water.

Learn more about ounces

brainly.com/question/29374025

#SPJ11

Misty needs 216 square inches of metal



to make a yield sign. If the height of the sign is 18 inches,



how long is the top edge of the sign?






24 inches




12 inches




198 inches




22 inches


pls give explanation not just the awnser

Answers

The answer is 96 inches, which is equivalent to 8 feet.

Find out the length of the top edge of the yeild sign ?

To find the length of the top edge of the yield sign, we need to use the formula for the area of a trapezoid:

A = (b1 + b2)h/2

where A is the area of the trapezoid, h is the height, b1, and b2 are the lengths of the two parallel bases of the trapezoid.

In this case, we are given the area of the sign (216 square inches) and the height (18 inches), but we don't know the length of either base. However, we do know that the shape of a yield sign is that of a regular octagon, which means it has eight equal sides and eight equal angles.

If we draw a line from the top of the sign to the midpoint of one of the sides, we will form a right triangle with the height of the sign as one leg, half the length of the top edge as the other leg, and the length of one of the sides as the hypotenuse. We can use the Pythagorean theorem to find the length of the side:

a^2 + b^2 = c^2

where a is the height of the sign (18 inches), b is half the length of the top edge (what we are trying to find), and c is the length of one of the sides.

Since the sign has eight sides, we can divide the total area by 8 to get the area of one of the eight triangles that make up the sign. We can then use this area to find the length of one of the sides:

A = (bh)/2

216 sq. in. = (bh)/2

432 sq. in. = bh

Since the sign is a regular octagon, each of the eight triangles has the same base (the side of the octagon) and height (half the length of the top edge), so we can use this equation to solve for b:

432 sq. in. = b(18 in.)/2

b = 48 in.

Now we know that half the length of the top edge is 48 inches, so the full length of the top edge is:

2(48 in.) = 96 in.

Learn more about Signs

brainly.com/question/14304253

#SPJ11

Amy borrows $1,000 on a simple interest loan. She pays an annual rate of 3. 5%. She will take 3 years to pay back the loan. How much interest will Amy pay?

Answers

The amount of interest Amy will pay over the 3 years is $105.

Simple interest is a method of calculating the interest amount on a loan or investment by multiplying the principal amount, the annual interest rate, and the time in years. In Amy's case, she borrowed $1,000 with an annual interest rate of 3.5% and will take 3 years to pay back the loan.

To calculate the interest Amy will pay, use the formula: Interest = Principal x Rate x Time

Interest = $1,000 x 0.035 (3.5% as a decimal) x 3 years

Interest = $1,000 x 0.035 x 3 = $105

Amy will pay $105 in interest over the 3 years.

Learn more about Simple interest here: https://brainly.com/question/25845758

#SPJ11

Benjamin went shopping for a new phone


because of a sale. The price on the tag was


$28, but Benjamin paid $15. 40 before tax.


Find the percent discount

Answers

The percent discount on the phone because of a sale is 45%.

To find the percent discount, we need to calculate the difference between the original price and the discounted price, and then express that difference as a percentage of the original price.

First, let's find the difference between the two prices: $28 - $15.40 = $12.60. This means that Benjamin saved $12.60 on the phone.

Now, let's find the percent discount. We can do this by dividing the savings by the original price, and then multiplying the result by 100: ($12.60 / $28) * 100 = 45%.

So, Benjamin received a 45% discount on the phone before tax. This calculation shows that the sale allowed him to save a significant amount on his purchase. It's important to compare original and discounted prices to determine if a sale provides good value.

Learn more about discount here: https://brainly.com/question/27519306

#SPJ11

What is the difference in credit score between those over the age of 55 and those between 18-24 years old?

Answers

According to recent studies, there is a notable difference in credit scores between those over the age of 55 and those between the ages of 18-24.

On average, individuals over the age of 55 tend to have higher credit scores than those in the 18-24 age range. This is primarily due to the fact that older individuals have had more time to establish and build their credit history, whereas younger individuals are just starting out and may not have had the opportunity to establish credit yet.

Additionally, older individuals tend to have more stable financial situations and may have less debt compared to younger individuals who may be dealing with student loans or other types of debt.

However, it's important to note that credit scores can vary greatly between individuals, regardless of age, and there are many factors that contribute to credit score, such as payment history, credit utilization, and length of credit history.

To know more about credit score:

https://brainly.com/question/29548283

#SPJ11

Jamie jogged x km. Anabel jogged 1/4 less than Jamie. Choose the equation that best represents the situation. A

y = 3/4x

b

x = 4/3y

c

y = 1/4x

d

x = 1/4y

Answers

The equation that best represents the situation is y = 3/4x. The correct answer is A.

The given problem involves two people, Jamie and Anabel, who jogged a certain distance. Let's say Jamie jogged x km. Anabel jogged 1/4 less than Jamie, which means she jogged 3/4 of x km (since 1 - 1/4 = 3/4).

To represent this situation in an equation, we need to find the relationship between the distance jogged by Jamie and the distance jogged by Anabel. Since Anabel jogged 3/4 of the distance jogged by Jamie, we can write:

distance jogged by Anabel = 3/4(distance jogged by Jamie)

Using the given variable x for the distance jogged by Jamie, we can rewrite the equation as:

distance jogged by Anabel = 3/4x

And since the question is asking for an equation that best represents the situation, the correct answer is:

Cy = 3/4x

Therefore, Cy = 3/4x is the equation that best represents the situation where Jamie jogged x km and Anabel jogged 1/4 less than Jamie. The correct answer is A.

Learn more abou equation at https://brainly.com/question/27358888

#SPJ11

There are 39 students in a class 22 offer maths,14 offer physics,and 16 offer biology if 5 offer both biology and math 7 offer at least one of the subject where every student offer at least one of the subjects. Find​

Answers

Where every student offer at least one of the subjects, there are 4 students who offer all three subjects.

To find the number of students who offer only one subject, we can use the formula:

number of students offering only one subject = total number of students offering the subject - number of students offering both subjects

For math, there are 22 students offering math and 5 offering both math and biology, so the number of students offering math only is:

22 - 5 = 17

Similarly, for physics and biology, the numbers of students offering the subjects only are:

14 - 7 = 7 (physics)
16 - 5 = 11 (biology)

To find the number of students offering all three subjects, we can use the formula:

number of students offering all three subjects = total number of students offering at least one subject - number of students offering only one subject in each subject + number of students offering no subject

We know that there are 7 students offering at least one subject. To find the number of students offering no subject, we can subtract this from the total number of students:

39 - 7 = 32

Now we can plug in the numbers:

number of students offering all three subjects = 7 - 17 - 7 - 11 + 32
= 4

Therefore, there are 4 students who offer all three subjects.

More on subjects: https://brainly.com/question/16295210

#SPJ11

Subtract.3 1/3−5enter your answer as a simplified mixed number by filling in the boxes.

Answers

The result of subtracting 5 from 3 1/3 is -2 2/3.

To subtract 5 from 3 1/3, we need to first convert the mixed number to an improper fraction. This can be done by multiplying the whole number (3) by the denominator of the fraction (3), and adding the numerator (1) to get 10/3. Therefore, 3 1/3 is equivalent to 10/3.

Next, we can subtract 5 from 10/3 by finding a common denominator of 3, which gives 15/3 - 10/3 = 5/3. This is the result in improper fraction form.

To convert back to a mixed number, we can divide the numerator (5) by the denominator (3), which gives a quotient of 1 and a remainder of 2. Therefore, the answer is -2 2/3.

For more questions like Subtract click the link below:

https://brainly.com/question/2346316

#SPJ11

Choose the function table that matches the given rule. Output = Input – 3 (1 point) Input Output –2 –5 1 –2 6 3 Input Output 2 –1 –2 3 0 –6 Input Output 5 2 2 –5 0 9 Input Output 6 3 –6 –3 5 0

Answers

The function table that matches the given rule output = input - 3 is

Input = -2, 1, 6 and output = -5, -2, 3

A) first function table

Output = Input - 3

Value of input:- -2

Putting the value of the input

Output = -2 -3

The value of output we get

Output = -5

Value of input:- 1

Putting the value of the input

Output = 1 -3

The value of output we get

Output = -2

Value of input:- 6

Putting the value of the input

Output = 6 -3

The value of output we get

Output = 3

Hence function table A is correct match

To know more about function click here :

https://brainly.com/question/12431044

#SJP1

PLSSS HELP.


Apples are on sale at a grocery store for per pound. Casey bought apples and used a coupon for off her purchase. Her total was. How many pounds of apples did Casey buy?



Part A: Write an equation that represents the problem. Define any variables.



Part B: Solve the equation from Part A. Show all work.



Part C: Explain what the solution to the equation represents

Answers

A: An equation that represents the problem is 1.75x - 0.45 = 4.45. B: Solving the equation from Part A gives x = 2.8. C: The solution to the equation represents the number of pounds of apple bought by Casey.

Part A: Write an equation that represents the problem. Define any variables.

Let x represent the number of pounds of apples Casey bought. The cost of apples is $1.75 per pound, so the total cost before using the coupon would be 1.75x. After using the $0.45 coupon, her total was $4.45. The equation representing this situation is:

1.75x - 0.45 = 4.45

Part B: Solve the equation from Part A.

Now, let's solve the equation:

1.75x - 0.45 = 4.45

Add 0.45 to both sides:

1.75x = 4.90

Now, divide both sides by 1.75:

x = 4.90 / 1.75

x = 2.8

Part C: Explain what the solution to the equation represents

The solution, x = 2.8, represents that Casey bought 2.8 pounds of apples at the grocery store.

Note: The question is incomplete. The complete question probably is: Apples are on sale at a grocery store for $1.75 per pound. Casey bought apples and used a coupon for $0.45 off her purchase. Her total was $4.45. How many pounds of apples did Casey buy? Part A: Write an equation that represents the problem. Define any variables. Part B: Solve the equation from Part A. Show all work. Part C: Explain what the solution to the equation represents.

Learn more about Equation:

https://brainly.com/question/27887972

#SPJ11

The following data points represent the number of holes
that moths ate in each of grandma marion's dresses.
7,8,8, 5, 7,8
using this data, create a frequency table.
number of holes
number of dresses
5
6
7
8

Answers

A frequency table was created using data points representing the number of holes in each of Grandma Marion's dresses. The table shows the number of dresses with 5, 7, and 8 holes, respectively.

To create a frequency table for the given data, first, the unique values in the data set are identified, which are 5, 7, and 8. Then, the number of occurrences of each unique value is counted, resulting in the frequencies 1 for 5, 2 for 7, and 3 for 8.

Count the frequency of each data point

5: 1 dress

7: 2 dresses

8: 3 dresses

Finally, these values are organized into a table with two columns, one for the unique values and another for their corresponding frequencies. The resulting frequency table shows the number of dresses with each number of holes eaten by moths.

To know more about frequency table:

https://brainly.com/question/31189964

#SPJ4

The value of a professional basketball player's autograph rose 40% in the last year. It is now worth $350.00. What was it worth a year ago? A. $260.00 B. $250.00 C. $270.00 D. $230.00

Answers

Answer: B

Step-by-step explanation: 250 x 140% = 350

a strawberry field, you will find 4 plants per square foot. How many strawberry plants will you find in a square field that has a length of 208 ft (approx 1 acre )?

Answers

Answer: 832 plants

Step-by-step explanation:

If there are 4 plants for every 1 square ft the ratio is 4:1.

This tells us to multiply 208x4 giving us 832.

Write and simplify the integral that gives the arc length of the following curve on the given integral. b. If necessary, use technology to evaluate or approximate the integral. y = -x² -5 on [-1,2]

Answers

To find the arc length of the curve y = -x² -5 on the interval [-1,2], we use the formula to evaluate:

L = ∫√(1 + (dy/dx)²) dx

where dy/dx is the derivative of y with respect to x.

First, we find dy/dx:

dy/dx = -2x

Next, we substitute dy/dx into the formula and simplify:

L = ∫√(1 + (-2x)²) dx
L = ∫√(1 + 4x²) dx

To evaluate this integral, we can use a trigonometric substitution. Let x = (1/2)tanθ, then dx = (1/2)sec²θ dθ. Substituting, we get:

L = ∫√(1 + 4(1/2)²tan²θ)(1/2)sec²θ dθ
L = (1/2)∫sec³θ dθ

To integrate sec³θ, we use integration by parts:

u = secθ, du/dθ = secθ tanθ
dv/dθ = sec²θ, v = tanθ

∫sec³θ dθ = secθ tanθ - ∫tan²θ secθ dθ
= secθ tanθ - ∫secθ dθ + ∫sec³θ dθ

Rearranging, we get:

2∫sec³θ dθ = secθ tanθ + ln|secθ + tanθ|

Therefore:

L = (1/2)(secθ tanθ + ln|secθ + tanθ|) + C

To evaluate L on the interval [-1,2], we need to find θ when x = -1 and x = 2. Using the substitution x = (1/2)tanθ:

When x = -1, θ = -π/4
When x = 2, θ = π/3

Substituting these values into the equation for L and simplifying, we get:

L = (1/2)(2√2 + ln(3 + 2√3) + π/4)

Therefore, the integral that gives the arc length of the curve y = -x² -5 on the interval [-1,2] is:

L = (1/2)(2√2 + ln(3 + 2√3) + π/4)

Note: If technology is used to evaluate or approximate the integral, the answer may differ slightly due to rounding errors.

MORE RELATED QUESTIONS on integrals: https://brainly.com/question/27419605

#SPJ11

The surface area of the square pyramid is 84 square inches. The side length of the base is 6 what is the value of x

Answers

With the surface area of the square pyramid 84 square inches and side length of the base is 6, the value of x is 4 inches, by assuming x as the slant height of the square pyramid.

Assuming that x refers to the slant height of the square pyramid, we can use the formula for the surface area of a square pyramid to solve for x:

Surface area of a square pyramid = base area + (0.5 x perimeter of base x slant height)

Since the base of the square pyramid is a square with side length 6,

the base area is 6² = 36 square inches.

The perimeter of the base is 4 times the side length, so it is 4 x 6 = 24 inches.

Substituting these values into the formula and simplifying, we get:

84 = 36 + (0.5 x 24 x x)

84 - 36 = 12x

48 = 12x

x = 4

Therefore, the value of x, the slant height of the square pyramid, is 4 inches.

To learn more about surface area : https://brainly.com/question/16519513

#SPJ11

Describe and correct the error in finding the circumference of ⊙C

Answers

Step-by-step explanation:

C= 2πr

Given,

Diameter= 9

so, radius = 9÷2 = 4.5

C= 2 x π x 4.5

= 28.3 (3.s.f)

By comparison a car with one of the worst car depreciations is a BMW 7 series. In 5 years it losses 72.6% of its value. If brand new the car costs $86,000, how much will the car be worth in 8 years?

Answers

The value of the car after 8 years is given as follows:

$10,836.76.

How to define an exponential function?

An exponential function has the definition presented as follows:

[tex]y = ab^\frac{x}{n}[/tex]

In which the parameters are given as follows:

a is the value of y when x = 0.b is the rate of change.n is the time needed for the rate of change.

The parameter values for this problem are given as follows:

a = 86000, n = 5, b = 1 - 0.726 = 0.274.

Hence the function for the value of the car after x years is given as follows:

[tex]y = ab^\frac{x}{n}[/tex]

[tex]y = 86000(0.274)^\frac{x}{5}[/tex]

The value of the car after 8 years is then given as follows:

[tex]y = 86000(0.274)^\frac{8}{5}[/tex]

y = $10,836.76.

More can be learned about exponential functions at brainly.com/question/2456547

#SPJ1

Help with problem in photo

Answers

Answer:

(x-6)²+(y+3)²=17²

Step-by-step explanation:

The equation of a circle with center (a,b) and radius r is given by the formula:

(x - a)² + (y - b)² = r²

In this case, the center of the circle is (6,-3), and it passes through the point (-9,5). To find the radius of the circle, we need to calculate the distance between the center and the point on the circle. Using the distance formula, we get:

r = √[(x₂ - x₁)² + (y₂ - y₁)²]

= √[(-9 - 6)² + (5 - (-3))²]

= √[225 + 64]

= √289

= 17

So, the radius of the circle is 17. Now we can plug in the values for the center and radius into the equation of a circle:

(x - 6)² + (y + 3)² = 17²

A binomial experiment has the given number of trials n and the given success probability p.


n=10, p=0. 2



(a) Determine the probability P (2 or fewer). Round the answer to at least three decimal places.


P(2 or fewer)

Answers

The probability P(2 or fewer) is 0.678.

To find the probability of 2 or fewer successes in a binomial experiment with 10 trials and a success probability of 0.2, we can use the binomial probability formula:

P(2 or fewer) = P(0) + P(1) + P(2)

where P(0), P(1), and P(2) represent the probabilities of getting 0, 1, or 2 successes, respectively.

P(0) = (10 choose 0) * 0.2^0 * 0.8^10 = 0.1074
P(1) = (10 choose 1) * 0.2^1 * 0.8^9 = 0.2684
P(2) = (10 choose 2) * 0.2^2 * 0.8^8 = 0.3020

Therefore,

P(2 or fewer) = 0.1074 + 0.2684 + 0.3020 = 0.6778

Rounded to at least three decimal places, the probability P(2 or fewer) is 0.678.

To learn more about Probability

https://brainly.com/question/24870672

#SPJ11

Height of 10th grade boys is normally distributed with a mean of 63. 5 in. And a standard deviation of 2. 9 in. The area greater than the z-score is the probability that a randomly selected 14-year old boy exceeds 70 in. What is the probability that a randomly selected 10th grade boy exceeds 70 in. ?Use your standard normal table.




Heights for 16-year-old boys are normally distributed with a mean of 68. 3 in. And a standard deviation of 2. 9 in. Find the z-score associated with the 96th percentile. Find the height of a 16-year-old boy in the 96th percentile. State your answer to the nearest inch

Answers

The probability that a randomly selected 10th grade boy exceeds 70 in is approximately 0.0127 or 1.27%.

The height of a 16-year-old boy in the 96th percentile is approximately 73 inches.

For the first question, we need to find the z-score for a height of 70 inches using the formula:

z = (x - μ) / σ

where x is the height of 70 inches, μ is the mean of 63.5 inches, and σ is the standard deviation of 2.9 inches.

z = (70 - 63.5) / 2.9 = 2.241

Using a standard normal table, we can find the area to the right of this z-score, which represents the probability that a randomly selected 10th grade boy exceeds 70 inches. The area to the right of 2.24 is 0.0127. Therefore, the probability is approximately 0.0127 or 1.27%.

For the second question, we need to find the z-score associated with the 96th percentile using a standard normal table. The 96th percentile is the point below which 96% of the data falls and above which 4% of the data falls. This corresponds to a z-score of approximately 1.75.

To find the height of a 16-year-old boy in the 96th percentile, we can use the formula:

x = μ + z * σ

where x is the height we want to find, μ is the mean of 68.3 inches, σ is the standard deviation of 2.9 inches, and z is the z-score we just found.

x = 68.3 + 1.75 * 2.9 = 73.28

Therefore, the height of a 16-year-old boy in the 96th percentile is approximately 73 inches.

To know more about probability, refer to the link below:

https://brainly.com/question/27372639#

#SPJ11

Jocelyn is considering taking out one of the two following loans. loan h is a three-year loan with a principal of $5,650 and an interest rate of 12.24%, compounded monthly. loan i is a four-year loan with a principal of $6,830 and an interest rate of 10.97%, compounded monthly. which loan will have the smaller monthly payment, and how much smaller will it be? round all dollar values to the nearest cent. a. loan h's monthly payment will be $42.46 smaller than loan i's. b. loan h's monthly payment will be $140.79 smaller than loan i's. c. loan i's monthly payment will be $11.88 smaller than loan h's. d. loan i's monthly payment will be $26.98 smaller than loan h's.

Answers

Loan H's monthly payment will be $42.46 smaller than loan I's (rounded to the nearest cent). The correct option is a.

To determine which loan will have the smaller monthly payment, we need to calculate the monthly payments for both loans using the given information.

For loan H, the monthly interest rate is 12.24%/12 = 1.02%, and the number of payments is 3 years x 12 months/year = 36. Using the formula for the monthly payment on a loan with monthly compounding, we have:

P = (r(PV))/(1 - (1+r[tex])^{(-n)})[/tex]

where P is the monthly payment, r is the monthly interest rate, PV is the principal value of the loan, and n is the total number of payments.

Plugging in the values given for loan H, we get:

P = (0.0102 x $5,650) / (1 - (1+0.0102)⁻³⁶) = $186.25

For loan I, the monthly interest rate is 10.97%/12 = 0.9142%, and the number of payments is 4 years x 12 months/year = 48. Using the same formula as above, we have:

P = (0.009142 x $6,830) / (1 - (1+0.009142)⁻⁴⁸) = $227.04

Therefore, the monthly payment for loan H is $186.25 and the monthly payment for loan I is $227.04.

To find the difference between the monthly payments, we subtract the monthly payment for loan H from the monthly payment for loan I:

$227.04 - $186.25 = $40.79

Therefore, the answer is (a) loan H's monthly payment will be $42.46 smaller than loan I's (rounded to the nearest cent).

To know more about monthly payment, refer to the link below:

https://brainly.com/question/2053367#

#SPJ11

Express tan H as a fraction in simplest terms.
F
H
28
7
G

Answers

Answer:

4 or [tex]\frac{4}{1}[/tex]

Step-by-step explanation:

To solve this we need to remember SOH-CAH-TOA. With SOH being Sine, CAH being Cosine, and TOA being Tangent. In the last term (TOA), the O means opposite and the A is adjacent. This means the segment opposite of angle H you have to divide that by the segment adjacent to H.

In this case, the opposite is 28 and the adjacent is 7. So we have to do [tex]\frac{28}{7}[/tex]. This is tan(H). Now we have to simplify this. Now we get our tangent of H to be [tex]\frac{4}{1}[/tex] or 4. So 4/1 or 4 is our answer

Curtis loves Pokémon! He went to school on Thursday and traded a bunch of cards to get new ones. He saw Dino and traded 3 of his cards for one of Dino's. Then a girl he liked, Tippi, wanted to trade cards. He was really nice to her because he liked her, so he traded 5 of his cards for 2 of hers. He then put his cards away. When he got home he noticed that 10 of his cards were missing. He was so upset that his mom bought him another pack of 12 cards. He hid half of his cards at home and took the rest to school the next day. He traded ¼ of the cards he brought to school to Dino again and got back 3 of Dino's cards. Curtis now has 9 cards at school. How many cards did he start with? How many cards total does he have now?

Answers

Curtis started with 84 cards and now has 12 cards at home and 9 cards at school, for a total of 21 cards.

How to find cards?

To find how many card ,We see Curtis has 9 cards at school after trading with Dino again, which means he had 12 cards before the trade.

Before his mom bought him another pack of 12 cards, he had 10 missing, so he must have had 24 cards in total (12 + 12).

He hid half of his cards at home, so he has 12 cards at home.

He traded ¼ of the cards he brought to school to Dino and got back 3 of Dino's cards. Let's call the number of cards he brought to school "x".

So, he traded x/4 cards to Dino, and got back 3 cards, which means he now has (x/4) - 3 cards.

We know that he now has 9 cards at school, so we can set up an equation:

(x/4) - 3 = 9

Solving for x, we get:

x/4 = 12

x = 48

So, Curtis brought 48 cards to school, which means he started with 24 + 12 + 48 = 84 cards in total.

Therefore, Curtis started with 84 cards and now has 12 cards at home and 9 cards at school, for a total of 21 cards.

Learn more about Cards

brainly.com/question/31598744

#SPJ11

Determine the missing length in each right triangle using the Pythagorean theorem. Round the answer to the nearest tenth, if necessary

Answers

The evaluated missing length in right triangle by using the Pythagorean theorem is  9 yards under the condition given the triangle is a right triangle.

The Pythagoras theorem projects that in a right triangle, the square of the hypotenuse is equal to the sum of the square of the other two sides,
It is given to us that in a right triangle,
Hypotenuse = 15 yd
Perpendicular = 12 yd

Therefore, applying Pythagoras theorem;
Base² = 15² - 12²

Base² = 225 - 144

Base² = 81

Base = √81

Base = 9 yards

Hence, The missing length present in the right triangle by applying the Pythagorean theorem is,
9 yards

To learn more about  Pythagoras theorem
https://brainly.com/question/27997683
#SPJ4


The complete question is

Determine the missing length in each right triangle using the Pythagorean theorem. Round the answer to the nearest tenth, if necessary


Amanda wants to add 6732 and 4975 how can Amanda use mental math to add the numbers is Amanda answer correct explain

Answers

she can add each number one by one , and whenever she needs to carry numbers over. She can add them to the already existing numbers. she cna check with a calculator

Other Questions
The coordinates of the midpoints of the four sides of a square are S(-4, 11), Q(2, 5), U(-4, -1), and A(-10, 5).Determine the perimeter and area of the square.5O perimeter is 144 units; area is 48 square unitsO perimeter is 48 units; area is 144 square unitsO perimeter is 24/2 units; area is 72 square unitsO perimeter is 72 units; area is 24/2 square units Given this unit circle what is the value of x Which of the following is not a run-on sentence?O The average length for a feature film is between 80 and 100 minutes but there are some films that exceed 14 hours in length and one film that was 85hours long.O According to data collected throughout film history, the average length for a feature film is between 80 and 100 minutes, although some far exceed thestandard running time.O There are some movies that clock in at over 14 hours long even though the average movie is around 80 minutes but can run up to 120 minutes.O With an average film length of 80 to 100 minutes it can be surprising to discover that some movies have been 14 hours long and even 85 hours long. A body has moved from its initial position at original along x-axis to final position x=5m under the influence of a force given by f =(4x)i n. what is the work done by the force in moving the body from its initial position to its final position ? Price $15 12 9 6 3 Quantity Demanded per Month 1 2 3 5 7 Price $15 12 9 6 3 In the BOX what you would expect to see if you plotted the numbers on the graph. The quantit demand pe month dec 0 1 2 3 4 5 6 7 Quantity b. It costs Charlene $2.00 to ride to bus to the dog owners house each time she goes. What number of dog walks will give her the most profit? OrCamille has a can of soup in the pantry. The circular lid has a radius of 3 inches. What is the lid's area?Use 3. 14 for How is poverty related to other social problems, such as discrimination, immigration and crime? SOMEONE HELPP!! giving brainlist to anyone who answers The term mitosis comes from the Greek root mitos, which means "thread." How does this meaningrelate to the process of mitosis?Please put in your own words. (based on proakis and salehi) a normalized modulating signal m.(t) has a bandwidth of 30000 hz and a power content of 0.1 watt. the carrier a cos(27fct) has a power contnet of 50 watts. (a) if m. (t) modulates the carrier using ssb amplitude modulation, what is the bandwidth and the power content of the modulated signal ussb(t)? (b) if the modulation instead is dsb-sc, what is the answer of part (a)? (c) if the modulation instead is dsb-lc (or conventional am) with modulation index 0.75, what is the answer of part (a)? What is the rhyme scheme of Sonnet 4 by Edmund Spenser? Who are the proles?Question 4 options:the educated class of Oceaniathe enemies of Oceaniathe ruling class of Oceaniathe working class of Oceania When did the glory days of the American cowboy begin? People commit unethical acts as a result of lack of care. Citing illustrations from Incident in the Park by Meja Mwangi, write an essay to validate this statement. What does the underlined sentencereveal about Ruthie's relationshipto Reggie? How are the mountains portrayed differently in the passage from Streams to the River, River to the Sea and the passage from Lewis and Clarks Journey of Discovery? A dead-end statement is another name for a sentence that is too ____________ to be developed into an essay. Imagine you have been hired as a consultant to support a firm that wishes to expand its operations internationally. your first assignment is to explain to the ceo and their staff the importance of understanding the difference between domestic expansion and international expansion. for your initial discussion post, consider the following questions as a basis to your explanation to the ceo: a. why is foreign investment so different from domestic investment?b. what should c-level executives consider in expanding internationally, as compared to domestically?c. what types of risk mitigation techniques could you suggest to the executives so that the firm can be successful in the proposed expansion? You get a job as a nurse. Your salary for the first year is $33,500. You willreceive a 1.5% increase every year. If you could save your entire salary, howmuch money would you have in 4 years? Round to the nearest dollar. Why is constructive alignment an important building block of a good lesson?